0 Daumen
189 Aufrufe

Aufgabe:

 \( \lim\limits_{n\to\infty} \)  \( \frac { 2 n ^ { 3 } - 3 n ^ { 2 } } { 4 n ^ { 3 } + 2 } \) = \( \frac{1}{2} \)


Problem/Ansatz:

… Wie beweise ich diese Konvergenz?

Ich bin nun soweit, dass ich den Beweisanfang aufgeschrieben habe. Also:

Beweis: Sei ε>0.

             Wähle n0 =

             Sei n ∈ ℕ>n0 .

             Dann gilt: I \( \frac { 2 n ^ { 3 } - 3 n ^ { 2 } } { 4 n ^ { 3 } + 2 } \) - \( \frac{1}{2} \)  I

                            = I \( \frac { 1 - 3 n ^ { 2 } } { 4 n ^ { 3 } + 2 } \) I


Problem: An dieser Stelle kann ich nicht weiter kürzen und müsste das n0 einsetzen. Allerdings weiß ich nicht wie., da ich ja im zähler und Nenner ein n stehen habe, kann ich ja nicht einfach den bruch in einer nebenrchnung mit ε gleihsetzen und nach n0 auflösen.

Meine Idee wäre gewesen, dass ähnlich zu machen, wie bei konvergierenden Funktionen, bei denen im Zähler und Nenner noch eine Variable steht, integrale für die Variable aufzustellen und das n0 = min {} zu wählen. Jedoch führt mich dieser Ansatz auch in die Irre, da dies ja eine Folge ist und ich ohnehin kein Integral aufstellen kann oder dergleichen.

Hat jemand eine Idee, wie man in solchen Konvergenzfällen wie diesen vorgeht?

Avatar von

1 Antwort

0 Daumen
 
Beste Antwort

Nach meinen Berechnungen muss es im Zähler -1-3n2 heißen.
Davon abgesehen, darfst du den Bruch großzügig abschätzen, z.B. wie folgt:$$\left\vert\frac{2n^3-3n^2}{4n^3+2}-\frac12\right\vert=\frac{3n^2+1}{4n^3+2}<\frac{3n^2+n^2}{4n^3+0}=\frac{4n^2}{4n^3}=\frac1n.$$Wähle also ein \(n_0\in\mathbb N\) mit \(n_0>\large\frac1\varepsilon\).

Avatar von 3,6 k

Ein anderes Problem?

Stell deine Frage

Willkommen bei der Mathelounge! Stell deine Frage einfach und kostenlos

x
Made by a lovely community